LSAT and Law School Admissions Forum

Get expert LSAT preparation and law school admissions advice from PowerScore Test Preparation.

 Brook Miscoski
PowerScore Staff
  • PowerScore Staff
  • Posts: 418
  • Joined: Sep 13, 2018
|
#60941
Chian,

Please review the first sentence of the stimulus.

Critics claim that it is required that we can indicate how the public will benefit.

The scientist disagrees with that requirement, which is why (E) is the correct answer.

(A) says that public benefit is irrelevant. There are two reasons why that does not describe the scientist's position.

First, the scientist disagrees that public benefit is a requirement. He doesn't say it's irrelevant. For instance, the scientist could believe that public benefit is a sufficient reason, even though it's not a necessary reason.

Second, the scientist didn't say that public benefit is irrelevant. He only said that being able to specify the benefit is not a requirement. In other words, the disagreement between the critics and the scientist is about whether it's necessary to be extremely specific about how the public will benefit.
User avatar
 elite097
  • Posts: 18
  • Joined: Apr 09, 2023
|
#100795
I am still not clear with the argument. WHat does it mean? How exactly is the author disagreeing with the critics. Critics are saying that funding is justified provided there is public benefit. WHat does the public support or anything hae to do with that? Not able to draw any linkages
User avatar
 Hanin Abu Amara
PowerScore Staff
  • PowerScore Staff
  • Posts: 60
  • Joined: Mar 29, 2023
|
#100808
Hi!

Ok so we're working with two premises.
First, we know that some scientists believe that public funding is justified only if we know how the public will benefit

then the author claims that IF the critics are right then there would not be as much public support as it has.

The author disagrees with the critics because he claims that public support plays a role which means knowing how the public benefit isn't the ONLY criteria.

E is the right answer because fact that there is public support shows that we don't need a benefit to be known.
User avatar
 Jeff Wren
PowerScore Staff
  • PowerScore Staff
  • Posts: 385
  • Joined: Oct 19, 2022
|
#100822
Hi elite097,

While it's true that the scientist is disagreeing with the critics, it's probably more helpful to think of the stimulus as a set of facts rather than an argument, especially since it's a Must be True question.

Fact 1: The critics believe that: If continued public funding is justified, then it can be indicated how the public will benefit from the project.

(Note - It's important to note that this is just the critics' belief, not a straight up conditional fact. This comes into play later.)

Fact 2: If the critics were right about this (meaning the conditional statement above), then there would not have been tremendous support for the project.

The contrapositive would be:

If there was tremendous support for the project, then the critics were wrong about this.

(Note - you don't have to worry about WHY the tremendous support for the project proves that the critics were wrong. The scientist is stating for a fact that if the critics were right, then a particular outcome would have happened.)

Fact 3: There was tremendous support for the project. (This is indicated in the final sentence of the stimulus "that even its critics acknowledge.")

The third fact, combined with the contrapositive of fact 2, let's us conclude that the critics were wrong.

In other words, the indication of how the public will benefit from the project is not necessary to determine whether continued public funding is justified (i.e. what the critics claimed was necessary isn't actually necessary), which is what Answer E is stating.

Get the most out of your LSAT Prep Plus subscription.

Analyze and track your performance with our Testing and Analytics Package.